Difference between revisions of "2014 AMC 12B Problems/Problem 11"

(Problem)
(Solution)
Line 6: Line 6:
  
 
== Solution ==
 
== Solution ==
 +
The line through <math>Q</math> only meets the parabola if the system of equations <math>y = x^2</math> and $y -

Revision as of 20:07, 20 February 2014

Problem

Let $\mathcal P$ be the parabola with equation $y = x^2$ and let $Q = (20, 14)$. There are real numbers $r$ and $s$ such that the line through $Q$ with slope $m$ does not intersect $\mathcal P$ if and only if $r < m < s$. What is $r + s$?

\[\textbf{(A)}\ 1\qquad\textbf{(B)}\ 26\qquad\textbf{(C)}\ 40\qquad\textbf{(D)}}\ 52\qquad\textbf{(E)}\ 80\] (Error compiling LaTeX. Unknown error_msg)

Solution

The line through $Q$ only meets the parabola if the system of equations $y = x^2$ and $y -